Sei sulla pagina 1di 51

Telegram PT Group Discussion 2019 (Environment)

Q: Which of the following statements best describes biotic potential?


(a) It refers to the potential biomass of a population measured as dry weight per unit area.
(b) It refers to the possible functional roles a species can play in an ecosystem.
(c) It refers to the maximum reproductive capacity of an organism under optimum environmental
conditions.
(d) It refers to the minimum population of a species required to sustain a food web.

Answer: C

o Biotic potential, the maximum reproductive capacity of an organism under optimum environmental
conditions. It is often expressed as a proportional or percentage increase per year, as in the statement "The
human population increased by 3 percent last year." It can also be expressed as the time it takes for a
population to double in size (doubling time).

o Full expression of the biotic potential of an organism is restricted by environmental resistance, any factor
that inhibits the increase in number of the population. These factors include un-favorable climatic
conditions; lack of space, light, or a suitable substrate; deficiencies of necessary chemical compounds or
minerals; and the inhibiting effects of predators, parasites, disease organisms, or un-favorable genetic
changes.

Q: Consider the following pairs:


Population Interaction Feature
1. Mutualism: Both the species are benefited
2. Competition: Both the species are harmed
3. Commensalism: One species is harmed and the other is unaffected
4. Amensalism: One species is benefited and the other is unaffected
Which of the pairs given above is/are correctly matched?
(a) 1 and 2 only
(b) 3 and 4 only
(c) 1, 2 and 3 only
(d) 1, 2, 3 and 4

Answer: A
Mutualism is the interaction in which both the interacting species benefit, for example, the association
between fungi and the roots of higher plants. The fungi help the plant in the absorption of essential
nutrients from the soil while the plant in turn provides the fungi with energy-yielding carbohydrates. This
pair is correctly matched.
Competition is the interaction in which species compete with each other to utilise the limited available
resource. In this interaction both the species lose as they try to outdo each other, for example, the
herbivores animals compete with each other to access limited grazing resources. This pair is correctly
matched.

© APTI PLUS ACADEMY FOR CIVIL SERVICES


Commensalism is the interaction among species in which one species benefits and the other is neither
harmed nor benefited, for example, an orchid growing as an epiphyte on a mango branch. In this interaction
while orchid benefits, the mango tree remains unaffected. This pair is not correctly matched.
Amensalism is the interaction in which one species is harmed while the other is unaffected, for example,
some higher plants like black walnut secrete substances that inhibit the growth of or kill the nearby
competing plants. This pair is not correctly matched.

Q: Consider the following statements regarding ecological pyramids:


1. Pyramid of number and biomass can be both upright and inverted.
2. Pyramid of energy is always upright.
Which of the statements given above is/are correct?
(a) 1 only
(b) 2 only
(c) Both 1 and 2
(d) Neither 1 nor 2

Answer: C
Statement 1 is correct: Pyramid of number and biomass can be both upright and inverted. For example
Pyramid of numbers in forest and parasitic food chain is upright and inverted respectively. Pyramid of
biomass is upright in forest ecosystem because the biomass of producer is much more than herbivores or
consumer which outweigh the biomass of upcoming tropic level (carnivores). However in pond ecosystem,
pyramid of biomass is inverted since the producer biomass is less than primary and secondary consumer
(biomass of large fish is more than phytoplankton or small producers).

Statement 2 is correct: Pyramid of energy is always upright, can never be inverted, because when energy
flows from a particular trophic level to the next trophic level, some energy is always lost as heat at each
step. Each bar in the energy pyramid indicates the amount of energy present at each trophic level in a given
time or annually per unit area.

Q: Which among the following is/are examples of the adaptations by organisms in order to cope up with
their environment?
1. Capability of organisms to burrow into the soil
2. Thick layer of fat below the skins of animals in polar regions
3. Absence of leaves in desert plants
Select the correct answer using the code given below.
(a) 1 and 2 only
(b) 1 only
(c) 2 and 3 only
(d) 1, 2 and 3

Answer: D
Adaptation is any attribute of the organism (morphological, physiological, behavioural) that enables the
organism to survive and reproduce in its habitat.
Statement 1 is correct. Some species are capable of burrowing into the soil to hide and escape from the
above-ground heat.

© APTI PLUS ACADEMY FOR CIVIL SERVICES


Statement 2 is correct. Mammals from colder climates generally have shorter ears and limbs to minimize
heat loss. (This is called the Allen's Rule.) In the polar seas aquatic mammals like seals have a thick layer of
fat (blubber) below their skin that acts as an insulator and reduces loss of body heat.

Statement 3 is correct. Many desert plants have a thick cuticle on their leaf surfaces and have their stomata
arranged in deep pits to minimise water loss through transpiration. They also have a special photosynthetic
pathway (CAM) that enables their stomata to remain closed during day time. Some desert plants like
Opuntia, have no leaves they are reduced to spines and the photosynthetic function is taken over by the
flattened stems.

Q: For a chemical to undergo biomagnification, it must be:


1. short lived
2. mobile and biologically active
3. soluble in water
Select the correct answer using the code given below
(a) 1 and 2 only
(b) 2 only
(c) 3 only
(d) 1, 2 and 3

Answer: B
In order for biomagnification to occur, the pollutant must be:
1.long-lived
2.mobile
3. soluble in fats
4. biologically active
If a pollutant is short-lived, it will be broken down before it can become dangerous. If it is not mobile, it will
stay in one place and is unlikely to be taken up by organisms. If the pollutant is soluble in water it will be
excreted by the organism. Pollutants that dissolve in fats, however, may be retained for a long time. It is
traditional to measure the amount of pollutants in fatty tissues of organisms such as fish. In mammals, we
often test the milk produced by females, since the milk has a lot of fat in it and because the very young are
often more susceptible to damage from toxins (poisons).

Q: Which of the following is/are examples of Mutualism?


1. Algae and lichens.
2. Rhizobium bacteria and plants.
3. Coral polyps and Zooxanthellae
4. Leech and cattle
Select the correct answer using the code given below:
(a) 1 and 3 only
(b) 2 and 4 only
(c) 1, 2 and 3 only
(d) 1, 2, 3 and 4

Answer: C

© APTI PLUS ACADEMY FOR CIVIL SERVICES


 Leeches feeding on cattle is an example of Parasitism i.e. association between two different species
where the symbiont benefits and the host is harmed. All the others are examples of Mutualism in
which both the organisms benefit.
 Algae provides food and seeks water and protection from lichens.
 Rhizobium bacteria receives carbohydrates from plants and in reverse plants get nitrogen fixed by
bacteria.
 Coral polyps provide protection and supply raw materials and Zooxanthellae provides food to host.

Q: With reference to Keystone species, consider the following statements:


1. A Keystone species exhibits disproportionately large effect on its environment relative to its abundance.
2. Figs in tropical rainforests are keystone species as they produce large quantity of fruits.
Select the correct answer using the code given below.
(a) 1 only
(b) 2 only
(c) Both 1 and 2
(d) Neither 1 nor 2

Answer: C
o The species having much greater influence on community characteristics, relative to their low abundance
or biomass, are called keystone species. These species play a vital role in controlling the relative abundance
of other species. Removal of keystone species causes serious disruption in the functioning of the
community.

For example, in the tropical rainforests, the different species of figs are the keystone species as these
produce large quantity of fruits.These fruits are eaten by monkeys, birds, bats and other vertebrates. Thus,
by protecting the fig trees, the animals dependent on them are also conserved.

Q: Consider the following statements with respect to Food Chain :


1. In an aquatic ecosystem, more energy flows through grazing food chain than detritus food chain.
2. In a terrestrial ecosystem larger fraction of energy flow through detritus food chain than grazing food
chain.
3. Only 10% of energy is lost in energy flow across different trophic levels.
Which of the statements given above is/are correct?
(a) 1 and 2 only
(b) 3 only
(c) 2 and 3 only
(d) 1, 2 and 3

Answer: A
Statement 1 is correct. Grazing food chain begins with producers, present at the first trophic level. The plant
biomass is then consumed by herbivores, which in turn are consumed by a variety of carnivores. In an
aquatic ecosystem, grazing food chain is the major conduit for energy flow.
Statement 2 is correct. Detritus food chain begins with detritus such as dead bodies of animals or fallen
leaves, which are then broken down into simple, inorganic material by decomposers or detritivores. In a
terrestrial ecosystem, detritus food chain is the major conduit for energy flow. Unlike in grazing web, all sub
soil organisms are included in detritus food chain.

Statement 3 is incorrect because 90% of energy is lost in energy flow across different trophic levels. Only

© APTI PLUS ACADEMY FOR CIVIL SERVICES


10% is transferred to next tropic level.

Q: Species diversity enhances the stability of a given ecosystem because:


(a) the micro-climate of the ecosystem becomes more favourable and localized.
(b) it narrows the ecological niche of the species.
(c) it provides alternative channels for energy flow in food web.
(d) species diversity reduces the mean net primary productivity.

Answer: C
Species diversity is defined as the number and abundance of different species that occupy a location.
Species diversity increases the number of links in the food web.
There can be various reasons for the stability of the ecosystem because of species diversity:
o Increased food web diversity increases the resilience of the system to outside invasions of exotic
organisms and reduces the fluctuation in the population within a given ecosystem.
o A large number of interacting feeding links provides alternative channels for energy flow and thus is
generated a wide variety of adjustments of the population to environmental changes and stresses within the
ecosystem.
o Another reason is more stability of a natural ecosystem because as the community succession operates,
the homeostasis increases due to more protection available to the members of the community against
external environment change.

Q: Which of the following is/are considered as Autotrophs?


1. Green plants
2. Blue green Algae
3. Human beings
4. Cattle
Select the correct answer using the code given below.
(a) 1 only
(b) 1 and 2 only
(c ) 2, 3 and 4 only
(d) 1, 2 and 4 only

Answer: B
Autotrophs are the organisms, which produce their own food by fixing solar energy in the presence of
abiotic substances. Green plants and Blue green algae are considered as autotrophs because they produce
their own food whereas Human beings and cattle are considered as heterotrophic organism because they
consume the food which is produced by producers or autotrophs.

Q: Consider the following statements:


1. Bioaccumulation refers to increase in concentration of a toxin at successive trophic levels.
2. Biomagnification is the gradual increase in concentration of a chemical over time in an organism.
Which of the statements given above is/are correct?
(a) 1 only
(b) 2 only
(c) Both 1 and 2
(d) Neither 1 nor 2

© APTI PLUS ACADEMY FOR CIVIL SERVICES


Answer: D
Statement 1 is incorrect. Biomagnification refers to increase in concentration of a toxin at successive
trophic levels. The toxin gets accumulated in the body of an organism and is passed on to the next level.
Example: DDT and other heavy metals such as mercury, cadmium, etc.
Statement 2 is incorrect. Bioaccumulation is the process by which certain toxic substances (such as heavy
metals and polychlorinated biphenyls) accumulate and keep on accumulating in living organisms, posing a
threat to health, life, and to the environment.
Q: Which among the following is/are the sedimentary nutrient cycles?
1. Nitrogen cycle
2. Phosphorus cycle
3. Sulphur cycle
Select the correct answer using the code given below.
(a) 1 and 2 only
(b) 2 and 3 only
(c) 3 only
(d) 1, 2 and 3

Answer: B
The movement of nutrient elements through the various components of an ecosystem is called nutrient
cycling. Nutrient cycles are of two types: (a) gaseous and (b) sedimentary. The reservoir for gaseous type of
nutrient cycle exists in the atmosphere and for the sedimentary cycle the reservoir is located in Earth's crust.
Nitrogen cycle is a gaseous nutrient cycle. Phosphorus and Sulphur cycles are sedimentary nutrient cycles.

Q: Consider the following statements with reference to Primary Productivity of ecosystem:


1. Net primary productivity is Gross Primary Productivity minus amount of biomass consumed by the
primary consumers.
2. Primary productivity of the oceans is more than the land.
Which of the statements given above is/are correct?
(a) 1 only
(b) 2 only
(c) Both 1 and 2
(d) Neither 1 nor 2

Answer: D
Statement 1 is incorrect. Net Primary Productivity is Gross Primary Productivity minus respiration losses.
Statement 2 is incorrect. The annual net primary productivity of the whole biosphere is approximately 170
billion tons. Of this, despite occupying about 70 per cent of the surface, the productivity of the oceans are
only 55 billion tons. Rest 115 biilion tons, is on land.

Q: With reference to ecological succession, the term Seral stage refers to


(a) the intermediate stage in an ecological succession.
(b) the final stage in the ecological succession that is in near equilibrium with the environment.
(c) the first stage of primary succession in an area.
(d) the first stage of secondary succession in areas where natural biotic communities have been destroyed .

Answer: A

© APTI PLUS ACADEMY FOR CIVIL SERVICES


A seral community (or sere) is an intermediate stage found in ecological succession in an ecosystem
advancing towards its climax community. The final community in ecological succession that is in near
equilibrium with the environment is called a climax community.

Q: Consider following statements regarding ecological pyramids:


1. Saprophytes are not given any place in ecological pyramids.
2. Both food chain and food web can be represented in an Ecological pyramid.
Which of the statements given above is /are correct?
(a) 1 only
(b) 2 only
(c) Both 1 and 2
(d) Neither 1 nor 2

Answer: A
Statement 1 is correct: saprophytes are not given any place in ecological pyramids even though they play
a vital role in the ecosystem.
Statement 2 is incorrect. Ecological pyramids do not accommodate a food web. It does not take into account
the same species belonging to two or more trophic levels. It assumes a simple food chain, something that
almost never exists in nature.

Q: Which of the following best defines the gradual change in certain characteristics exhibited by
communities along with gradual change in one or more environmental gradients?
(a) Ecotone
(b) Ecotype
(c) Ecocline
(d) Ecological Niche

Answer: C
o Ecocline (ecological gradient) A gradation from one ecosystem to another when there is no sharp
boundary between the two. It is the joint expression of associated community (coenocline) and complex
environmental gradients.
o Ecotone: An ecotone is a transition area between two biomes. It is where two communities meet and
integrate. It may be narrow or wide, and it may be local (the zone between a field and forest) or regional
(the transition between forest and grassland ecosystems).

o Ecotype: In evolutionary ecology, an ecotype, sometimes called ecospecies, describes a genetically distinct
geographic variety, population or race within a species, which is adapted to specific environmental
conditions. They typically show morphological differences.

o An ecological niche is the role and position a species has in its environment; how it meets its needs for
food and shelter, how it survives, and how it reproduces. A species' niche includes all of its interactions with
the biotic and abiotic factors of its environment. No two species have identical niches

Q. What is the correct order of the level of organization of ecology?


a) Individual, Community, Population, Ecosystem, Biome and Biosphere.
b) Individual, Community, Population, Biome, Ecosystem and Biosphere.

© APTI PLUS ACADEMY FOR CIVIL SERVICES


c) Individual, Population, Community, Ecosystem, Biome and Biosphere.
d) Individual, Population, Community, Biome, Biosphere and Ecosystem.

Ans) c
Explanation: The main levels of organisation of ecology are six. In ascending order, they follow as:
Individual, Population, Community, Ecosystem, Biome and Biosphere

Q: Consider the following pairs:


Biome type Vegetation type
1. Taiga: No trees, except stunted shrubs
2. Savannah: Grasses with scattered trees and fire resisting thorny shrubs
3. Temperate: Broad-leaved trees with less diversity of plant species
Which of the pairs given above is/are correctly matched?
(a) 1 and 3 only
(b) 2 and 3 only
(c) 1 and 2 only
(d) 2 only

B
Pair 1: In the higher latitudes (50°-70°) of Northern hemisphere, the Coniferous forests are found. These
are also called as Taiga. These forests are also seen in the higher altitudes.
In Tundra type of vegetation, the growth of natural vegetation is very limited. Only mosses, lichens and
very small shrubs are found here. It grows during the very short summer. This is called Tundra type of
vegetation. This vegetation is found in the polar areas of Europe, Asia and North America. Thus, pair 1 is
incorrectly matched.
Pair 2: Tropical grasslands: These grow on either side of the equator and extend till the tropics. This
vegetation grows in the areas of moderate to low amount of rainfall. They can grow very tall, about 3 to 4
metres in height. Savannah grasslands of Africa are of this type. Savannah shrubs are fire-resistent which
helps them survive periodic, natural fire spread in forests. Thus, pair 2 is correctly matched.
Pair 3: Temperate forests have moderately dense broad-leaved trees and have less diversity of plant
species. Oak, Beach, Maple etc. are some common species. Thus, pair 3 is correctly matched.

Q: Which of the following are the favorable conditions for Coral formations?
1. Shallow continental shelf
2. High sediments in the water
3. Areas where fresh waters from rivers enter oceans
4. Mean annual temperatures of above 22 degree C.
Select the correct answer using the code given below.
(a) 1 and 2 only
(b) 1, 3 and 4 only
(c) 2 and 3 only
(d) 1 and 4 only

Answer:D
1 is correct. Corals need to grow in shallow water where sunlight can reach them. Corals depend on the
zooxanthellae (algae) that grow inside of them for oxygen and other things, and since these algae needs
sunlight to survive, corals also need sunlight to survive. Corals rarely develop in water deeper than 50
meters.

© APTI PLUS ACADEMY FOR CIVIL SERVICES


2 is incorrect. Corals need clear water that lets sunlight through; they don't thrive well when the water is
opaque. Sediment and plankton can cloud water, which decreases the amount of sunlight that reaches the
zooxanthellae.
3 is incorrect. Corals need saltwater to survive and require a certain balance in the ratio of salt to water.
This is why corals don't live in areas where rivers drain fresh water into the ocean.
4 is correct. Reef-building corals require warm water conditions to survive. Different corals living in different
regions can withstand various temperature fluctuations. However, corals generally live in water
temperatures of 20–32° C.

Q: Which of the following is known as the Rain forests of the oceans?


(a) Mangroves
(b) Coral reefs
(c) Sea grass
(d) Oysters

Answer: B
Coral reefs are also called rainforests of ocean. About 1,000,000 species inhabits the corals reefs which is
more diverse than rainforests.

Q:With reference to Eutrophication, consider the following statements:


1. It is the process of decrease of nutrients in the water bodies.
2. It leads to the depletion of oxygen from water bodies.
3. It results in growth of harmful algal blooms.
Which of the statements given above is/are correct?
(a) 1 only
(b) 1 and 2 only
(c) 2 and 3 only
(d) 1, 2 and 3

Answer: C
Statement 1 is incorrect. Eutrophication arises from the oversupply of nutrients (mainly phosphates), which
leads to over growth of plants and algae. After such organisms die, the bacterial degradation of their
biomass consumes the oxygen in the water.

Statement 2 is correct. Eutrophication occurs both naturally and man-made. It is the response to the
addition of nutrients such as nitrates and phosphates through discharge of domestic sewage, industrial
effluents and fertilizers from agriculture fields or naturally. This process decreases the dissolved oxygen in
the water body.
Statement 3 is correct. The nutrients serve as rich food for algae leading to algal blooms.

Q:Consider the following statements with regard to Marine ecosystems:


1. The marine waters are divided into Photic and Aphotic zones based on the salinity of water.
2. Green plants are found only in Photic zones.
3. Aphotic zone is completely devoid of life.
Which of the statements given above is/are correct?
(a) 1 only
(b) 2 only
(c) 2 and 3 only

© APTI PLUS ACADEMY FOR CIVIL SERVICES


(d) 1 and 2 only

Answer: B
Statement 1 is incorrect. The marine waters are divided into Photic and Aphotic zones based on the
availability of sunlight. Photic zone is the surface layer of the ocean that receives sunlight. The uppermost
80 m (260 feet) or more of the ocean, which is sufficiently illuminated to permit photosynthesis by
phytoplankton and plants, is called the euphotic zone. Sunlight insufficient for photosynthesis illuminates
the disphotic zone, which extends from the base of the euphotic zone to about 200 m. The thicknesses of
the photic and euphotic zones vary with the intensity of sunlight as a function of season and latitude and
with the degree of water turbidity. Presence of sunlight supports growth of plants and animals. Green plants
are found only in photic zones since they need sunlight for photosynthesis. Hence statement 2 is correct.

Statement 3 is incorrect. Though the bottom most, or aphotic, zone is the region of perpetual darkness that
lies beneath the photic zone and includes most of the ocean waters, it does have non-photosynthetic plants,
bacteria and fungi.

Q: Which of the following statements is/are correct regarding freshwater ecosystems?


1. Lotic ecosystems are still water ecosystems whereas Lentic ecosystems are running water ecosystems.
2. Thermocline in an aquatic ecosystem is the zone of constant temperature.
Select the correct answer using the code given below:
(a) 1 only
(b) 2 only
(c) Both 1 and 2
(d) Neither 1 nor 2

Answer: D
Freshwater ecosystems can be divided into two types:
1. Standing water or lentic (lenis, meaning calm) ecosystems like a lake, pond, marshes, or swamps. These
are also known as pond ecosystems.
2. Running water or lotic ecosystems are that of a river or Spring.
Hence statement 1 is incorrect.
Statement 2 is incorrect. Thermocline is the zone of aquatic ecosystem in which temperature of the water
changes rapidly.

Q: Consider the following statements regarding Benthic Zone of an ocean ecosystem:


1. It is the region extending between high tide and low tide levels.
2. This is the most productive zone in Ocean due to high nutrient supply.
3. Most of the organisms found here are luminescent.
Which of the statements given above is/are true?
(a) 3 only
(b) 1 and 2 only
(c) 1 and 3 only
(d) 1, 2 and 3

© APTI PLUS ACADEMY FOR CIVIL SERVICES


Answer: A
Statement 1 is incorrect: Benthic zone refers to the floor of the ocean. Production, in this zone, is limited by
low nutrient supply thus 2nd statement is incorrect.
Statement 3 is correct. Most of the organisms found here are luminescent. Rooted organisms are sea lilies,
sea fan, sponges, etc. Snails and clams are embedded in the mud whereas star fish and sea urchins move on
the surface.

Q: With reference to cold water corals, which of the following statements is/are correct?
1. They do not require Zooxanthellae to survive.
2. Worldwide distribution of cold water coral reef is greater than tropical reef.
Select the correct answer using the code given below.
(a) 1 only
(b) 2 only
(c) Both 1 and 2
(d) Neither 1 nor 2

Answer: C
Cold -water corals extend to deeper, darker parts of the oceans than tropical corals, ranging from near the
surface to the abyss, beyond 2,000 metres where water temperatures may be as cold as 4 °C. They inhabit
deep water, not shallow water. Like tropical corals, they provide habitat to other species, but deep-water
corals do not require zooxanthellae to survive (statement 1 is correct).
United Nations Environment Program reports that there are more cold water coral reefs worldwide than
tropical reefs. Their extent is much larger than their tropical counterpart. Some, as the various individual
reefs stretching from Norway as far south as West Africa, are when combined far bigger than more famous
tropical ones such as Australia’s Great Barrier Reef. (statement 2 is correct).

Q: Which of the following are the sources of renewable energy?


1. Geo-thermal energy
2. Biofuels
3. Biomass energy
4. Nuclear energy
Select the correct answer using the codes given below
(a) 1 and 3 only
(b) 1, 2 and 3 only
(c) 2 and 4 only
(d) 1, 2, 3 and 4

Answer: B
Renewable source of energy: Renewable sources are those which can be generated continuously in nature
and are inexhaustible. Following are the renewable sources:
o solar energy
o Geo-thermal energy (statement 1 correct)
o Tidal energy
o Wind energy
o Hydropower
o Biomass energy (statement 3 correct)
o Bio-fuels (statement 2 correct)
o Hydrogen

© APTI PLUS ACADEMY FOR CIVIL SERVICES


Following are the non-renewable sources
o fossil fuels (coal, petroleum etc)
o Uranium (statement 4 incorrect)
o Thorium

Q: Which of the following is/are the characteristics of Compressed Natural Gas (CNG)?
1. It is lead free
2. It is a liquid fuel like LPG
3. On combustion it emits lesser Carbon monoxide than petrol
4. The noise level of CNG engine is higher than that of diesel
Select the correct answer using the code given below.
(a) 1 and 2 only
(b) 1 and 3 only
(c) 2, 3 and 4 only
(d) 1, 3 and 4 only

Answer: B
CNG is an acronym for Compressed Natural Gas. It is a mixture of hydrocarbons consisting of approximately
80 to 90 per cent methane in gaseous form.
Statement 1 is correct. CNG is lead free and reduces harmful emissions. Another advantage of CNG is the
extension of life of lubricating oils as the fuel does not contaminate and dilute the crankcase oil.
Statement 2 is incorrect. It is not a liquid fuel and is not the same as LPG (Liquified Petroleum Gas).
Statement 3 is correct. Compared to petrol or diesel CNG vehicles emit 40% less of nitrous oxide, 80% less of
Carbon monoxide and 25% less of Carbon dioxide.
Statement 4 is incorrect. Noise level of a CNG engine is much lower than that of a diesel engine.

Q:Which of the following projects uses 'Run of the river systems' for the purpose of generating electricity?
1. Tidal energy project
2. Small-scale hydro power project
3. Large reservoir based hydro power project
Select the correct answer using code given below.
(a) 1 and 2 only
(b) 2 and 3 only
(c) 2 only
(d) 1and 3 Only

Answer: C
Statement 1 is incorrect: Tidal energy uses the periodic rise and fall of waters of the ocean for generation of
electricity and not run of the river system.

Statement 2 is correct: Small-scale hydro project: The flow of river water is harnessed by channelising it
through a narrow canal without any creation of artificial reservoir, this type of system is called as 'run of the
river systems'. It produces small amount of electricity which can be utilized on a small scale.

© APTI PLUS ACADEMY FOR CIVIL SERVICES


Statement 3 is incorrect: Large reservoir based hydro project: It involves creation of artificial reservoir of
water to produce a huge amount of electricity. Run-of-river systems do not rely on large storage reservoirs,
but rather on the diverted river water to drive turbines, and then discharge the water back into the river
system.

Q: Consider the following pairs:


Alternative energy source – Method of utilization
1. Ipomoea plants – Biomass gasification
2. Municipal soild waste - Incineration
3. Banana leaves - Pellitisation
Which of the pairs given above is/are correctly matched?
(a) 2 only
(b) 1 and 3 only
(c) 2 and 3 only
(d) 1, 2 and 3

Answer: D
All the given pairs are correctly matched.
1. Gasification refers to the technique of converting biomass into a combustible gas. Ipomoea plants
(creeping plants) are grown around water bodies. A new technique of Biomass gasification for generating
electricity from Ipomoea has been developed in Madhya Pradesh.
2. The most common method of energy generation from Municipal solid waste is Incineration. In simple
terms it stands for a type of waste treatment process, where the waste collected are burnt at high
temperatures.
3. The Centre of Biochemical Technology (CBT), New Delhi has developed a method of generating fuel from
the banana leaves. This method is known as pellitisation. Banana leaves are crushed into powder. Then clay
or cowdung is added to the powder to bind the mixture. This mixture is then converted into pellets. These
pellets can be used as fuel and have a good calorific value.

Q: "Methanol has been promoted as an alternative transportation fuel from time to time over the past forty
years". Which of the following statements regarding Methanol is not correct?
(a) It can be produced from natural gas.
(b) It is a biodegradable fuel.
(c) Similar to hydrocarbon fuels, it is not soluble in water.
(d) It can be blended with ethanol and gasoline.

Answer: C
Option (c) is not correct.
Methanol, like ethanol but unlike hydrocarbon fuels, is water soluble. As a consequence, it is transported
through diffusion and convection through the environment at much faster rate than hydrocarbons. In
addition to fast migration, methanol bio-degrades much faster than hydrocarbons. Fast transport and
degradation result in short lifetimes. Hydrocarbons, on the other hand, comprise many stable compounds,
which include some, such as aromatics, that are highly toxic to bio-organisms and degrade slowly.

© APTI PLUS ACADEMY FOR CIVIL SERVICES


Methanol has attractive features for use in transportation:
It is a liquid fuel which can be blended with gasoline and ethanol and can be used with today‟s vehicle
technology at minimal incremental costs.
It is a high octane fuel with combustion characteristics that allow engines specifically designed for
methanol fuel to match the best efficiency of diesels while meeting current pollutant emission regulations.
Produced from renewable biomass, methanol is an attractive green house gas reduction transportation
fuel option in the longer term.
Multiple ways exi st for introduction of methanol into the fuel infrastructure (light blends or heavy blends)
and into vehicles (light duty or heavy duty applications). The optimal approaches are different in different
countries and in different markets.
Large scale produ ction of methanol from natural gas and coal is a well developed technology.
It is a safe fuel. The toxicity (mortality) is comparable to or better than gasoline. It also
biodegrades
quickly (compared to petroleum fuels) in case of a spill.

Q: Off-grid Renewable Energy can be a game changer as it can provide energy to isolated communities and
areas. Which of the following can be used as a part of such system?
1. Biomass gasifiers
2. Micro hydro projects
3. Solar Photo Voltaic Roof-top Systems
Select the correct answer using the code below.
(a) 1 and 2 only
(b) 2 and 3 only
(c) 1 and 3 only
(d) 1, 2 and 3

Answer: D
Distributed/decentralized renewable power projects using wind energy, biomass energy, hydro power and
hybrid systems are being established in the country to meet the energy requirements of isolated
communities and areas which are not likely to be electrified in near future.
Off-grid Renewable Energy/ Power can be generated using the following:
Biomass based heat and power projects and industrial waste to-energy projects for meeting captive needs
 Biomass gasifiers for rural and industrial energy applications
 Watermills/micro hydro projects – for meeting electricity requirement of remote villages
Small Wind Energy & Hybrid Systems - for mechanical and electrical applications, mainly where grid
electricity is not available.
 Solar PV Roof-top Systems for abatement of diesel for power generation in urban areas
The Ministry of New and Renewable energy is running a program with main objectives of: supporting R&D to
make such systems more reliable and cost effective, demonstration, field testing, strengthening
manufacturing base.

Q: Consider the following statements about Bio-gas:


1. It is a combination of methane and carbon dioxide.
2. It is formed by anaerobic digestion of organic materials.
3. It normally burns without smoke.
Which of the statements given above is/are correct?
(a) 1 and 2 only

© APTI PLUS ACADEMY FOR CIVIL SERVICES


(b) 1 and 3 only
(c) 2 and 3 only
(d) 1, 2 and 3

Answer: D
 Statement 1 is correct: Bio-gas contains up to 65% methane and 35% CO2.
 Statement 2 is correct: Biogas is produced by the anaerobic digestion of organic materials in a sealed
fermenter. This fermenter transforms organic materials into biogas by using methane-producing bacteria
through a biologically complex process at approximately 38 - 55 degrees Celsius.
 Statement 3 is correct: It burns without smoke, leaves no residue like ash in wood, charcoal and coal
burning. Its heating capacity is high. The large-scale utilisation of bio-waste and sewage material provides a
safe and efficient method of waste-disposal besides supplying energy and manure. Bio-gas is also used for
lighting.

Q: Which of the following can be used in the production of second generation ethanol to power motor
vehicles?
1. Rice straw
2. Jatropha
3. Bamboo
Select the correct answer using the code given below.
(a) 2 only
(b) 1 and 2 only
(c) 1 and 3 only
(d) 1, 2 and 3

Answer: C
First generation biofuels are made from the sugars and vegetable oils found in arable crops, which can be
easily extracted using conventional technology.
In comparison, second generation biofuels are made from lignocellulosic biomass or woody crops,
agricultural residues or waste, which makes it harder to extract the required fuel.
Second generation ethanol draws on previously unused (ligno -) cellulosic plant parts, such as straw or
corn cobs. These can be made from bamboo, rice straw, wheat straw, cotton straw etc to power vehicles.
Jatropha oil is mainly converted into biodiesel for use in diesel engines, not ethanol.

Q: Consider the following statements about thermal pollution in a closed water body:
1. It reduces the oxygen holding capacity of the water body
2. It hampers the growth of blue green algae
3. It causes death of fishes in the aquatic system
Which of the following statements given above is/are correct?
(a) 1 only
(b) 2 and 3 only
(c) 1 and 3 only
(d) 1, 2 and 3

Answer: C

© APTI PLUS ACADEMY FOR CIVIL SERVICES


o Thermal Pollution: When the discharged water that is released back into the water body is around 10
degrees celcius higher than the temperature of the water body it is termed as thermal pollution.
o The rise in temperature reduces the oxygen holding capacity of the water body, as oxygen carrying
capacity of warm water is less than cold water.
o This causes deaths or migration of several fish species like trout etc
o However, blue green algae growth enhances in the warm water. They produce a toxin which further
hampers the biodiversity.

Q: Consider the following statements with regard to carbon monoxide gas:


1. It is a foul odour gas which smells like rotten egg.
2. It is produced as a result of incomplete combustion of carbon.
3. Paddy field is an important source of carbon monoxide.
Which of the statements given above is/are correct?
(a) 1 and 3 only
(b) 2 only
(c) 1, 2 and 3
(d) 2 and 3 only

Answer: B
Statement 1 is incorrect. Carbon Monoxide is a colourless and odourless gas.
Statement 2 is correct. It is produced as a result of incomplete combustion of carbon.
Statement 3 is incorrect. Paddy field is an important source of Methane and not Carbon Monoxide.

Q:Which of the following pollutants are present in the motor vehicle exhaust emissions?
1. Carbon Dioxide
2. Carbon Monoxide
3. Nitrous Oxide
4. Hydrocarbons
Select the correct answer using the code given below:
(a) 1, 2 and 4 only
(b) 2, 3 and 4 only
(c) 1 and 3 only
(d) 1, 2, 3 and 4

Answer: D
All the by-products mentioned above are part of exhaust emissions produced by motor vehicles. Passenger
vehicles are a major pollution contributor, producing significant amounts of nitrogen oxides, carbon
monoxide, and other pollution.
An exhaust gas is a gas emitted through a combustion process. The exhaust gas is actually a combination of
many different gases: N2, CO2, H2O and O2. Though some are harmless, there are few that are harmful and
are considered major pollutants. One of the most dangerous of these is CO (carbon monoxide).

Q: Consider the following statements with regard to Bio-chemical Oxygen Demand (BOD):
1. It is the amount of oxygen needed by aerobic micro organisms to break down organic material present in
a given water sample.
2. Higher level of BOD indicates lower pollution level of the water.

© APTI PLUS ACADEMY FOR CIVIL SERVICES


Which of the statements given above is/are correct?
(a) 1 only
(b) 2 only
(c) Both 1 and 2
(d) Neither 1 nor 2

Answer: A
Statement 1 is correct Biochemical oxygen demand is the amount of dissolved oxygen needed (i. e.,
demanded) by aerobic micro organisms to break down organic material present in a given water sample at
certain temperature over a specific time period.
Statement 2 is incorrect. Higher the BOD, higher is the pollution level in the water. The amount of BOD in
the water is an indicator of the level of pollution. If too much organic matter is added to the water all the
available oxygen is used up. This causes fish and other forms of oxygen dependent aquatic life to die.

Q: Which of the following are non-point sources of pollution in rivers?


1. Surface drains carrying municipal sewage
2. Run-off from agricultural fields
3. Dumping of un-burnt/half-burnt dead bodies
4. Dhobi ghats
5. Effluents from industries
Select the correct answer using the code given below.
(a) 1, 4 and 5 only
(b) 2 and 3 only
(c) 2, 3 and 4 only
(d) 1, 2, 3 and 4 only

Answer: C
o Point sources: These are organized sources of pollution where the pollution load can be measured, e.g.
surface drains carrying municipal sewage or industrial effluents, sewage pumping stations and sewerage
systems, trade effluents from industries, etc. Pollution loads due to untreated sewage is one of the main
reasons threatening the ecological health of rivers. Most of the urban lakes in the country are also facing
similar challenges.
o Non-point sources: These are non-measurable sources of pollution such as run-off from agricultural fields
carrying chemicals and fertilizers, run-off from solid waste dumps and areas used for open defecation,
dumping of un-burnt/half-burnt dead bodies and animal carcasses, dhobi ghats, cattle wallowing, etc.

Q: Which of the following are the sources of Methane?


1. Biomass burning
2. Landfills
3. Rice fields
4. Coal mining
5. Permafrost
Select the correct answer using the code given below:

© APTI PLUS ACADEMY FOR CIVIL SERVICES


(a) 3 and 4 only
(b) 1, 3 and 4 only
(c) 1, 2 and 5 only
(d) 1, 2, 3, 4 and 5

Answer: D
There are many sources which are responsible for the emission of Methane, a major GHG.
Source Methane : release rate Per cent
1 Natural wetlands 21.3
2 Rice fields 20.4
3 Biomass burning 10.2
4 Enteric fermentation 14.8
5 Termites 7.4
6 Landfills 7.4
7 Gas drilling 8.3
8 Coal mining 6.5
9 Oceans 1.9
10 Other sources (permafrost) 1.8

Q: Which of the following is/are constituents of fly ash?


1. Black soot
2. Lead
3. Cadmium
4. Nickel
Select the correct answer using the codes given below.
(a) 1 only
(b) 2, 3 and 4 only
(c) 1, 2 and 3 only
(d) 1, 2, 3 and 4

Answer: D
o Fly ash: also know as Pulverised fuel ash is the light weight substance which tend to fly around in hot flue
gases.
o It includes black soot occurring due to combustion of coal, vapourised particles of lead, cadmium, nickel
etc.
o Fly ash is actively being used in concrete to enhance its strength. However, it negatively impacts the
environment by depositing over snow, thus reducing the albedo and initiating melting. Himalayan ecology in
particular is negatively impacted by fly ash.

Q: With reference to the role of various gases in the atmosphere, consider the following statements:
1. Ozone in troposphere is harmful for human beings.
2. Nitrogen is used for manufacture of fertilizers.
3. Carbon dioxide absorbs infra-red rays and keeps the earth warm.
Which of the statements given above is/are correct?
(a) 1 and 2 only
(b) 1 and 3 only
(c) 3 only
(d) 1, 2 and 3

© APTI PLUS ACADEMY FOR CIVIL SERVICES


Answer: D
Statement 1 is correct. While stratospheric ozone shields human beings from ultraviolet radiation, the
tropospheric ozone is harmful. This irritating, reactive molecule damages forests and crops; destroys nylon,
rubber, and other materials; and injures or destroys living tissue.
Statement 2 is correct. Nitrogen is largely used as nitrogen fertilizers in agriculture. Nitrogen fertilizers are
manufactured on a large scale from the nitrogen contained in the atmosphere.
Statement 3 is correct. Carbon dioxide present in the air absorbs the infrared rays and keeps the earth
warm. If carbon dioxide is absent in the air, these infrared rays would escape into the atmosphere and the
earth would cool down very rapidly to a very low temperature.
Carbon dioxide, the quantity of which is small in the atmosphere is important for plants. Plants use carbon
dioxide as a raw material for the manufacture of food by the process of photosynthesis and release oxygen
as a by-product.
Q: Which of the following pairs is/are correctly matched?
Pollutant Source
1. Organo-chlorine : Paper industry
2. Lead : Paints
3. Sulphur dioxide : Thermal power plants
Select the correct answer using the code given below.
(a) 1 and 2 only
(b) 1 and 3 only
(c) 2 only
(d) 1, 2 and 3

Answer: D
Pair 1 is correct. Chlorine is used as a bleaching agent in the paper industry for whitening of the paper. It
reacts with the organic content in the cellulose and produces organo-chlorine, which is a toxic compound
and causes water-pollution. Bleaching with Oxygen or peroxide is a cleaner alternative to chlorine.

Pair 2 is correct. The most common sources of lead pollution in environment, are manufacturing plants and
industries that make wide use of lead in the production of paint, batteries, bearings, solders, seals and a
wide range of electronic appliances such as TV glass, TV tubes, ballasts, weights, foils, wires, crystal glass etc.

Pair 3 is correct. Burning of Sulphur in coal or heavy oil in thermal power plants release SO2 in the air.
SO2 combines with oxygen and water to form Sulphuric acid.

Q: Which of the following statements is not correct?


(a) Pollutants cannot easily escape to the stratosphere from the troposphere because of temperature
inversion.
(b) Depletion of the ozone layer at the south pole is facilitated by the presence of ice crystals in the clouds.
(c) Nitrogen oxides derived from fertilizers is an ozone depleting agent.
(d) None of the above

Answer: D
Statement A is correct. The Ozone layer absorbs the incident Ultraviolet Rays received by the sun, which
leads to the heating-up of the stratosphere and the resultant temperature inversion. Due to the

© APTI PLUS ACADEMY FOR CIVIL SERVICES


temperature inversion, it becomes difficult for the pollutants from the troposphere to escape to the
stratosphere.
Statement B is correct. The extremely cold (-400C) conditions in the South Pole facilitate the formation of
ice crystals in the Polar Stratospheric clouds. These ice crystals act as catalysts, facilitating the conversion
of Hydrogen Chloride gas into Chlorine and Hydrogen. The Chlorine radicals react with the Ozone,
freeing the Oxygen molecules and depleting the Ozone layer.

Statement C is correct. Nitrous oxide breaks down when it reaches the stratosphere to form other gases,
called nitrogen oxides, that trigger ozone-destroying reactions.

Q: Consider the following statements regarding Photochemical Smog:


1. It occurs in cool humid climate.
2. It result from the action of sunlight on unsaturated hydrocarbons and nitrogen oxides.
3. It causes corrosion of metals and building materials.
Which of the statements given above is/are correct?
(a) 1 and 3 only
(b) 2 and 3 only
(c) 2 only
(d) 1, 2 and 3

Answer: B
Statement 1 is incorrect. The word smog is derived from smoke and fog. There are two types of smog:
Photochemical smog occurs in warm, dry and sunny climate. The main components of the photochemical
smog result from the action of sunlight on unsaturated hydrocarbons and nitrogen oxides produced by
automobiles and factories. (Statement 2 is correct.)

It also causes corrosion of metals, stones, building materials, rubber and painted surfaces. (Statement 3 is
correct.)

Q: Which of the following is/are the emissions from coal combustion at thermal power plants?
1. Mercury
2. Particulate matter
3. Oxides of Sulphur
Select the correct answer using the code given below.
(a) 1 only
(b) 2 and 3 only
(c) 1 and 3 only
(d) 1, 2 and 3

Answer: D
Coal fired thermal power plants are one of the main contributors for atmospheric pollution and greenhouse
gases. Emissions that come from these plants could be categorized into three different categories:
Gaseous emissions Carbon Dioxide, Carbon Monoxide, Sulphur Dioxide and Nitrogen Dioxide which lead
to global warming and acid rain.
 Particulate matter emissions - this fine dust that is emanated from the stacks of power plants is a health
hazard.
Trace elements like Mercury, Cadmium and Lead which are also health hazards.

© APTI PLUS ACADEMY FOR CIVIL SERVICES


Q: Which of the following wetland is represented by as many as three protected areas, namely a National
Park, Wildlife Sanctuary and Marine Sanctuary?
a) Vembanad Lake
b) Nalsarovar
c) Kolleru
d) Bhitarkanika

Answer: d
Explanation: Bhitarkanika wetland is represented by as many as three protected areas, namely a National
Park, Wildlife Sanctuary and Marine Sanctuary.

Q: Consider the following statements with reference to a National Park:


1. It has moist deciduous, semi evergreen, evergreen, shola forests and montane grasslands.
2. It is home to largest population of Lion Tailed Macaque.
3. It comes under Project Elephant.
Which of the following National Parks is characterised by the above features?
(a) Manas
(b) Bannerghata
(c) Silent valley
(d) Nandadevi

Answer: C
Silent Valley National Park has exceptional Ecosystem diversity from moist deciduous, semi evergreen,
evergreen and shola forests to montane grasslands. It has largest population of Lion Tailed Macaque
and Nilgiri Langur. It comes under Project Elephant area.
Manas national park is ruled out as Shola forests are endemic to southern part of western ghats.
Bannerghata national park is located just south of Bengaluru and is deprived of shola forest.
Lion tailed macaque is not there in Nandadevi national park.

Q: Consider the following statements with regard to a national park:


1. It has the high density of tigers in the protected areas of world.
2. It is recognized as Important Bird Area by Birdlife International.
3. It is a UNESCO world heritage site.
Which of the following National Parks is characterised by the above features?
(a) Silent valley
(b) Kaziranga
(c) Keoladeo
(d) Sariska

Answer: B
Kaziranga National Park - is also a world heritage site. The park is recognized as an Important Bird Area by
BirdLife International for the conservation of avifaunal species. The park hosts two-thirds of the world's
Great One-horned rhinoceros. Kaziranga also has the highest density of tigers among the protected areas in

© APTI PLUS ACADEMY FOR CIVIL SERVICES


the world and was declared a Tiger Reserve in 2006. It has tall elephant grass, marshland, and dense tropical
moist broadleaf forests.
Keoladeo does not have Tigers and is also not an mportant
I Bird Area. It is both a national park and a
world heritage site.
Sariska is not a world heritage site.

Q:Consider the following statements regarding Loktak lake:


1. It is known for phumdis floating over it.
2. It provides habitat to Sangai.
3. It is covered under Ramsar list and Montruex Record.
4. It is a brakish water lake.
Which of the statements given above is/are correct?
(a) 3 and 4 only
(b) 2 and 3 only
(c) 1, 2 and 4 only
(d) 1, 2 and 3 only

Answer: D
Loktak Lake is the largest freshwater lake in North -East India (Manipur) is famous for the phumdis
(heterogeneous mass of vegetation, soil, and organic matters at various stages of decomposition) floating
over it.
It is listed underRamsar as well as Montreux Record.
The Keibul mjaoLa National Park on the lake is the only remaining natural habitat of the endangered
brow-antlered deer or the dancing deer, locally known as Sangai. Keibul Lamjao National park is situated
inside the lake to conserve the species.

Q: Which of the following is a national park, a UNESCO world heritage site and a Ramsar site in India?
(a) Chilika Lake national park
(b) Manas national park
(c) Keoladeo national park
(d) Sundarbans national park

Answer: c

© APTI PLUS ACADEMY FOR CIVIL SERVICES


Q:Which of the following criteria must be met by a region to qualify as a biodiversity hotspot?
1. It must contain at least 1,500 species of vascular plants as endemics.
2. It must have lost at least 70% of its original habitat.
Select the correct answer using the code given below.
(a) 1 only
(b) 2 only
(c) Both 1 and 2
(d) Neither 1 nor 2

Answer: C
To qualify as a biodiversity hotspot, a region must meet two strict criteria:
It must have at least 1,500 vascular plants as endemics — which is to say, it must have a high percentage

© APTI PLUS ACADEMY FOR CIVIL SERVICES


of plant life found nowhere else on the planet. A hotspot, in other words, is irreplaceable.
it has to have lost at least 70% of its original habitat. In other words, it must be threatened.
Around the world, 35 areas qualify as hotspots. They represent just 2.3% of Earth's land surface, but they
support more than half of the world's plant species as endemics - i.e., species found no place else - and
nearly 43% of bird, mammal, reptile and amphibian species as endemics.

Q: Consider the following pairs:


Tiger reserve Location
1. Bandhavgarh : Chattisgarh
2. Valmiki : Uttar Pradesh
3. Satkosia : Odisha
Which of the pairs given above is/are correctly matched?
(a) 1 and 2 only
(b) 2 and 3 only
(c) 3 only
(d) 1, 2 and 3

Answer: C
 Bandhavgarh National Park is spread at vindhya hills in Madhya Pradesh. Bandhavgarh National Park
topography varies between steep ridges, undulating, forest and open meadows. Bandhavgarh National Park
is known for the Royal Bengal Tigers.
 Valmiki National Park and Wildlife Sanctuary is located in the West Champaran district of Bihar state,
India. Valmiki Tiger Reserve (VTR) is one of the natural virgin recesses in east India, situated in the north
west corner of Bihar. The pristine forest and wilderness of VTR is an excellent example of Himalayan Terai
landscape. VTR comprises the Valmiki National Park and Valmiki Wildlife Sanctuary.
 Satkosia Tiger reserve is located in Odisha. The Tiger Reserve supports moist deciduous forest, dry
deciduous forest and moist peninsular Sal forest. This area is the home for Tiger, Leopard, Elephant, Gaur,
Sambar, Spotted deer, Mouse deer, Nilgai, Chousingha, Sloth bear, Wild dog etc., Varieties of resident and
migratory birds, reptilian species (Gharial, Magar, Crocodile, Fresh Water turtle, Poisons & Non poisons
snakes etc.

Q: Consider the following pairs:


National Parks Located in
1. Betla : Chota Nagpur Plateau
2. Kanger Ghati : Aravalli Range
3. Guindy : Western Ghats
Which of the pairs given above is/are correctly matched?
(a) 1 only
(b) 2 and 3 only
(c) 1 and 3 only
(d) 1, 2 and 3

Answer: A
 Betla National Park is a national park located in the Chota Nagpur Plateau of the Palamu district of the
Indian state of Jharkhand, in India
 Kanger Valley National Park is situated near Jagdalpur, in the Bastar region of Chhattisgarh. Thus Aravali
is incorrect.

© APTI PLUS ACADEMY FOR CIVIL SERVICES


Guindy national park is located in Chennai. Thus western ghats is incorrect. It is the 8th smallest
National Park of India and one of the very few national parks situated inside a city.

Q: Consider the following pairs:


Protected Area River passing through
1. Bhitarkanika : Brahmani
2. Manas : Brahmaputra
3. Corbett : Ganga
Which of the pairs given above is/are correctly matched?
(a) 1 only
(b) 2 and 3 only
(c) 1 and 3 only
(d) 1, 2 and 3

Answer: A
 Manas river passes through the Manas National Park which is a tributary to the Brahmaputra.Two major
biomes are represented in Manas - the grassland biome and the forest biome. Manas is the only landscape
in the world where pristine Terai Grasslands are seen merging with the Bhabar grasslands interspersed with
diverse habitats ascending to Semi-Evergreen forests and then to Bhutan Himalayas.
The Biodiversity is very rich here. The last population of the Pygmy Hog survive in the wilds of Manas and
nowhere else in the worl
 Bhitarkanika is mainly confined to the deltaic regions of river Brahmani and Baitarani. It has much
significance with regard to ecological geomorphological and biological background which includes mangrove
forests, rivers, creeks, estuaries, back water, accreted land and mud flats. The Sanctuary comprises of
Mangrove Forests, meandering rivers, innumerable criss-crossed tidal inundated creeks provide last refuge
to the already endangered salt water crocodile (Crocodile porosus) which is the
flagship species. Besides estuarine crocodile, the Sanctuary is also rich in avifauna, mammalian and
reptilian population. The Ramganga river (West) along with its significant tributaries Sonanadi, Palain and
Mandal forms the
prominent hydrological resource for the Corbett. The river Kosi also flows by national park and serves
as the significant water resources for the nearby area.

Q: Which of the following is the correct sequence of Ramsar sites arranged from west to east?
1. Nalsarovar
2. Kolleru
3. Harike
4. Bhitarkanika
Select the correct answer using the code given below.
(a) 1- 2- 4- 3
(b) 1- 3- 2- 4
(c) 2- 1- 3- 4
(d) 1- 3- 4- 2

© APTI PLUS ACADEMY FOR CIVIL SERVICES


Answer: b

Q: Identify the national park based on below features.


1. It is a part of world’s 38 ‘hottest hotspots’ of biological diversity.
2. It is home to unique, threatened and endangered species like the lion-tailed macaques
3. Rivers Tunga, Bhadra and Netravati, originate from the heart of this National Park.
Select the correct answer using the codes given below.
a) Kudremukh National Park
b) Bandipur National Park
c) Mudumalai National Park
d) Silent Valley National Park

Answer: a
Explanation: The Kudremukh National Park in the Western Ghats is a part of the world’s 38 ‘hottest
hotspots’ of biological diversity, a UNESCO world heritage site.
It is more than 630 sq km area of montane grasslands and tropical rainforests form the largest protected
block within the Western Ghats.
The park is home to unique, threatened and endangered biodiversity including lion-tailed macaques, tigers,
Malabar civets and great pied hornbills.
The rivers Tunga, Bhadra and Netravati, which originate from the heart of the park, sustain millions of rural
people in south India.
The literary meaning of kuduremukh is ‘horse face’, and is in reference to the appearance of a mountain
peak located within the park.
The Park nurtures a diverse range of flora and fauna. There are highly dense forests rich in varieties of wild
life like as tiger, leopard, wild dog, Malabar giant squirrel, common langur, sloth bear, gaur, sambar, spotted
deer, barking deer and the sloth bear.
The other attraction of the place is, the temple of goddess bhagawathi, and a varaha image 1.8m. The river
bhadra and the tunga freely flow through the parklands. Kadambi waterfalls is also the point of interest for
the travelers.

Q: Which of the following species are critically endangered?


1. Namdapha Flying Squirrel

© APTI PLUS ACADEMY FOR CIVIL SERVICES


2. Malabar Civet
3. Siberian Crane
4. Wild Ass
Select the correct answer using the code given below.
(a) 1 and 4 only
(b) 1, 2, 3 only
(c) 2, 3 only
(d) 1, 3, 4 only

Answer: B
As per the latest (2011) quantitative aluation
ev done by the International Union for Conservation of
Nature (IUCN) there are 57 critically endangered species of animals in India. The Namdapha Flying
Squirrel, Malabar Civet, Siberian Crane are critically endangered.
Wild Ass Near
is Threatened animal not critically endangered.

Q: Which of the following statements is/are correct about a biosphere reserve?


1. A biosphere reserve focuses on protection of a single species rather than the entire ecosystem.
2. Human activity is completely prohibited in both core and buffer zones of a biosphere reserve.
Select the correct answer using the code given below.
(a) 1 only
(b) 2 only
(c) Both 1 and 2
(d) Neither 1 nor 2

Answer: D
Biosphere reserves are areas of terrestrial and coastal ecosystems promoting solutions to reconcile the
conservation of biodiversity with its sustainable use. It is divided into core, buffer and transition zone. The
human intervention is limited to buffer zone only. The transition zone is a zone of active cooperation
between management and local people.

Statement 1 is not correct. Unlike national parks and sanctuaries which focus on the habitat of some species
or on a single species, the biosphere reserve takes into consideration the entire ecosystem.

Statement 2 is not correct. Biosphere reserves are demarcated into following 3 inter-related zones:
Core Zone: Core zone must contain suitable habitat for numerous plant and animal species, including higher
order predators and may contain centres of endemism. Core areas often conserve the wild relatives of
economic species and also represent important genetic reservoirs having exceptional scientific interest. A
core zone being National Park or Sanctuary/protected/regulated mostly under the Wildlife (Protection) Act,
1972. Whilst realizing that perturbation is an ingredient of ecosystem functioning, the core zone is to be
kept free from human pressures external to the system.

Buffer Zone: The buffer zone, adjoins or surrounds core zone, uses and activities are managed in this area in
the ways that help in protection of core zone in its natural condition. These uses and activities include
restoration, demonstration sites for enhancing value addition to the resources, limited recreation, tourism,
fishing, grazing, etc; which are permitted to reduce its effect on core zone. Research and educational

© APTI PLUS ACADEMY FOR CIVIL SERVICES


activities are to be encouraged. Human activities, if natural within BR, are likely to continue if these do not
adversely affect the ecological diversity.

Transition Zone: The transition area is the outermost part of a biosphere reserve. This is usually not
delimited one and is a zone of cooperation where conservation knowledge and management skills are
applied and uses are managed in harmony with the purpose of the biosphere reserve. This includes
settlements, crop lands, managed forests and area for intensive recreation and other economic uses
characteristics of the region.

Q: Which among the following regions in India is/are biodiversity hotspots?


1. Eastern Ghats
2. Eastern Himalayas
3. Nicobar Islands
Select the correct answer using the code given below.
(a) 1 and 3 only
(b) 2 only
(c) 2 and 3 only
(d) 1, 2 and 3

Answer: C
According to Conservation international, to qualify as a hotspot a region must meet two strict criteria: it
must contain at least 1,500 species of vascular plants (> 0.5% of the world's total) as endemics, and it has to
have lost at least 70% of its original habitat.

Q: Which of the following leads to habitat destruction?


1. Construction activities in forest area
2. Introduction of invasive species
3. Volcanic eruption
Select the correct answer using the code given below.
(a) 1 only
(b) 1 and 3 only
(c) 2 and 3 only
(d) 1, 2 and 3

Answer: D
Habitat deterioration and destruction is the main cause of threat to many wild plant species and plant
communities. Habitat destruction could be caused as a result of:
o Clearing of forest areas for settlement or agricultural expansion, commercial lodgings, large hydel
schemes, fire, human and livestock pressure, etc.
o Pollution (both air and water) stresses ecosystem, mismanagement of industrial and agriculture wastes
threaten both terrestrial and aquatic ecosystem.
o Over exploitation, mainly for commercial purposes like meat, fur, hides, body organs, medicine, etc.
o Accidental or deliberate introduction of exotic species which can threaten native flora and fauna directly
by predation or by competition and also indirectly by altering the natural habitat or introducing diseases.

Q: Which among the following have been classified as extinct by the IUCN?
1. Dodo
2. Quagga

© APTI PLUS ACADEMY FOR CIVIL SERVICES


3. Bali tiger
Select the correct answer using the code given below.
(a) 1 only
(b) 1 and 3 only
(c) 2 and 3 only
(d) 1, 2 and 3

Answer: D
The IUCN Red List (2004) documented the extinction of 784 species (including 338 vertebrates, 359
invertebrates and 87 plants) in the last 500 years. Some examples of recent extinctions include the dodo
(Mauritius), quagga (Africa), thylacine (Australia), Steller's Sea Cow (Russia) and three subspecies (Bali,
Javan, Caspian) of tiger.

Q: The term Biomining refers to?


(a) Extraction of important information through analysis of genome
(b) Use of micro-organisms for extraction of ores from mines
(c) Extraction of metals from ores using micro-organisms
(d) Use of enzymes for commercial production of industrial chemicals

Answer: c

Biomining is the process of using microorganisms (microbes) to extract metals of economic interest from
rock ores or mine waste.

Q: Consider the following statements regarding Great Indian Bustard.

1. It is Critically Endangered on the IUCN Red List


2. Project Great Indian Bustard has been initiated by Gujarat government.

Choose the correct option from the options given below:


a. 1 Only
b. 2 Only
c. Both 1 and 2
d. None of the above
Answer: a
Rajasthan, home to one of the last remaining populations of the Great Indian Bustard, has charted out a
plan to recover the population of the critically endangered bird. On June 5, the state announced Rs 12 crore-
Project Great Indian Bustard, which is to be initiated from this year.

Q: Consider the following statements regarding Irrawaddy Dolphin:


1. They are found in both fresh waters and marine waters.
2. They are classified as 'Critically Endangered' in the IUCN Red List.
3. In India, they are found in Chilka Lake.
Which of the statements given above is/are correct?
(a) 1 only
(b) 1 and 3 only

© APTI PLUS ACADEMY FOR CIVIL SERVICES


(c) 2 and 3 only
(d) 1, 2 and 3

Answer: B
Irrawaddy Dolphins are species primarily found in Southeast Asian estuaries and mangrove areas, with
freshwater populations occurring in river systems. Therefore, they are found in both fresh waters and
marine waters. Hence statement 1 is correct.
They are classified as Endangered’
‘ in the IUCN Red List of Threatened Species. Statement 2 is wrong.
Freshwater subpopulations occur in the rive r Mahakam of Kalimantan (Indonesian Borneo), the
Ayeyarwady (formerly Irrawaddy) of Myanmar (formerly Burma), and the Mekong Delta of Laos, Cambodia,
and Vietnam. They are also reported in isolated brackish (saltwater and freshwater) water bodies, such as
Chilka Lake in India and Songkhla Lake in Thailand. Hence statement 3 is correct.

Q: Consider the following statements with regard to Olive Ridley turtles:


1. They are found in warm waters of the Pacific and Indian oceans.
2. They are herbivores and thrive exclusively on algae and sea grasses.
3. The coast of Odisha in India is the largest mass nesting site in the world.
Which of the statements given above is/are correct?
(a) 2 only
(b) 1 and 3 only
(c) 2 and 3 only
(d) 1, 2 and 3

Answer: B
Olive Rid ley turtle is the smallest and most abundant of all sea turtle found in the world. It gets its name
from its olive colored carapace, which is heart-shaped and rounded. It is found in warm waters of the Pacific
and Indian oceans. Hence statement 1 is correct.
The coast of Orissa in India is the largest mass nesting site for the Olive -ridley, followed by the coasts of
Mexico and Costa Rica. Hence statement 3 is correct.
Olive Ridley turtles are carnivores, and feed mainly on jellyfish, shrimp, snails, cra bs, molluscs and a
variety of fish and their eggs. Hence statement 2 is incorrect.

Q: Which among the following has been declared as the National Heritage Animal of India?
(a) Elephant
(b) Tiger
(c) Gangetic Dolphin
(d) Cow

Answer: A
India's Environm ent Ministry declared the elephant as the National Heritage Animal in 2010 in order to
increase protective measures for the country's nearly 29,000 elephants.
Declaring it the National Heritage Animal will give it due place as the emblem of ecological nsitivity.
se

Q: Consider the statements regarding the India Cooling Action Plan (ICAP)

© APTI PLUS ACADEMY FOR CIVIL SERVICES


1. India is the first country in the world to develop a document on ICAP
2. Ministry of Science has released the draft on ICAP.

Choose the correct option


a. 1 Only
b. 2 Only
c. Both 1 and 2
d. None of the above.

Answer: a

India is the first country in world to develop such a document on ICAP, which addresses cooling requirement
across sectors and lists out actions which can help reduce the cooling demand. So statement 1 is correct.
Ministry of Environment and Forest Climate Change (MoEFCC) has released the draft India Cooling Action
Plan (ICAP). ICAP has been prepared by the ministry after extensive deliberations and multi-stakeholders
engagement in public domain for receiving comments. So statement 2 is incorrect.

Q: Who among the following heads the National Board of Wildlife in India?
(a) The President
(b) The Prime Minister
(c) Minister for Environment, Forest andClimate Change
(d) Cabinet Secretary

Answer: B
The National Board for Wildlife is chaired by India's Prime Minister and its vice chairman is Minister for
Environment, Forest and Climate Change. Further, the board is mammoth body with 47-membersincluding
Parliament Members, NGOs, eminent conservationists, ecologists and environmentalists,Government
secretaries of various departments.
National Board for Wild Life is a Statutory Organization• constituted under the Wildlife Protection Act,
1972. Theoretically, the board is advisory in nature and advises the Central Government on framingpolicies
and measures for conservation of wildlife in the country. However, it is a very important bodybecause it
serves as apex body to review all wildlife-related matters and approve projects in and aroundnational parks
and sanctuaries.

Q: The National Green Tribunal can hear cases related to which of the following Acts?
1. The Water (Prevention and Control ofPollution) Act, 1974.
2. The Environment (Protection) Act,1986.
3. The Biological Diversity Act, 2002.
4. Wildlife (Protection) Act, 1972.
Select the correct answer using the codegiven below.
(a) 1, 2 and 3 only
(b) 2, 3 and 4 only
(c) 1 and 4 only
(d) 1, 2, 3 and 4

© APTI PLUS ACADEMY FOR CIVIL SERVICES


Answer: A
The NGT has the power to hear all civil cases relating to the implementation of laws listed in Schedule Iof the
NGT Act. These include the following:
The Water (Prevention and Control of Pollution) Act, 1974;
The Water (Prevention and Control of Pollution) Cess Act, 1977;
The Forest (Conservation) Act, 1980;
The Air (Prevention and Control of Pollution) Act, 1981;
The Environment (Protection) Act, 1986;
The Public Liability Insurance Act, 1991;
The Biological Diversity Act, 2002.
This means that any violations pertaining only to these laws can be challenged before the NGT. The NGT has
not been vested with powers to hear any matter relating to the Wildlife (Protection) Act, 1972, the
Indian Forest Act, 1927 and various laws enacted by States relating to forests, tree preservation etc.
Therefore, specific and substantial issues related to these laws cannot be raised before the NGT.

Q: Consider the following statements regardingCentral Pollution Control Board:


1. It was established under the Water (Prevention and Control of Pollution) Act, 1974.
2. It is chaired by the Minister of Environment, Forest and Climate Change.
3. It can impose penalties on any industryfound violating pollution norms.
Which of the statements given above is/arecorrect?
(a) 2 and 3 only
(b) 1 only
(c) 1, 2 and 3
(d) None

Answer: B
 Statement 1 is correct: Central Pollution Control Board has been established under Water (Preventionand
control of Pollution) Act, 1974.
 Statement 2 is not correct: It serves as a field formation and also provides technical services to
theMinistry of Environment and Forests of the provisions of the Environment (Protection) Act, 1986. It is
notchaired by the Minister of Environment, Forest and Climate Change but the chairman is nominated by
thecentral government.
 Statement 3 is not correct: The CPCB is only an advisory body and lacks power of imposing penalties.
The Central Board has not been granted any powers to force state governments to adopt uniform
standardsor to take over those cases where state governments are reluctant to enforce such standards.
While theCPCB is given an advisory role, the SPCBs have the critical function of ensuring compliance
andenforcement of pollution control activities.

Q: With reference to Animal Welfare Board of India, consider the following statements:
1. It is a statutory body created under Prevention of Cruelty to Animals Act, 1960.
2. It provides for financial assistance to Animal welfare organizations and NGOs.
Which of the statements given above is/arecorrect?
(a) 1 only
(b) 2 only
(c) Both 1 and 2
(d) Neither 1 nor 2

© APTI PLUS ACADEMY FOR CIVIL SERVICES


Answer: C
The Animal Welfare Board of India is a statutory advisory body on Animal Welfare Laws and
promotesanimal welfare in the country. Established in 1962 under Section 4 of the Prevention of Cruelty to
AnimalsAct, 1960, the Animal Welfare Board of India was started under the stewardship of Late Smt.
Rukmini Devi Arundale, well known humanitarian. Hence statement 1 is correct.

It’s one of the function is to give financial assistance and other assistance to Animal Welfare Organisations
functioning in any localarea or to encourage the formation of Animal Welfare Organisations in any local area
and NGO's. Hencestatement 2 is correct

Q: Consider the following statements about Intergovernmental Panel on Climate Change (IPCC)

1. The Intergovernmental Panel on Climate Change (IPCC) is the United Nations body for assessing the
science related to climate change.
2. The IPCC does not conduct its own research.
3. It was created by UNDP in 1988
Which of the above statements is/are correct?
a. 1 only
b. 1 and 2
c. 1and 3
d. 1, 2, 3

Solution: b)

The IPCC was created to provide policymakers with regular scientific assessments on climate change, its
implications and potential future risks, as well as to put forward adaptation and mitigation options.The
assessment reports are a key input into the international negotiations to tackle climate change. Created by
the United Nations Environment Programme (UN Environment) and the World Meteorological Organization
(WMO) in 1988, the IPCC has 195 members.

Q.) With reference to the Cartagena Protocol on Biosafety, consider the following statements:
1. It is an additional agreement to the Convention on Biological Diversity (CBD).
2. The protocol establishes procedures for regulating the import and export of living modified organism from
one country to another.
Which of the statement(s) given above is/are correct?
a) 1 only
b) 2 only
c) Both 1 and 2
d) Neither 1 nor 2

Answer: c

© APTI PLUS ACADEMY FOR CIVIL SERVICES


Explanation: Both statements are correct.
Cartagena Protocol on Biosafety to the Convention on Biological Diversity
 Protection in the field of the safe transfer handling use of living modified organism (LMO)

The convention clearly recognizes these twin aspects of modern biotechnology:


• Access to and transfer of technologies.
• Appropriate procedures to enhance the safety of biotechnology technologies.
• The protocol establishes procedures for regulating the import and export of living modified
organism from one country to another.

Q: With reference to the International Union for Conservation of Nature (IUCN), which of the following
statements is/are correct?
1. It includes both government and non government organisations as its members.
2. It has Observer Status at the UnitedNations (UN).
Select the correct answer using code givenbelow.
(a) 1 only
(b) 2 only
(c) Both 1 and 2
(d) Neither 1 nor 2

Answer: C
Go through the link : https://www.iucn.org/

o Statement 1 is correct. IUCN is the world's oldest and largest global environmental organisation. It has
more than 1,200 member organizations including 200+ government and 900+ non-government
organizations.
o IUCN's mission is to "influence, encourage and assist societies throughout the world to conserve
natureand to ensure that any use of natural resources is equitable and ecologically sustainable."
o Over the past decades, IUCN has widened its focus beyond conservation ecology and now incorporates
issues related to gender equality, poverty alleviation and sustainable business in its projects.
o Statement 2 is correct. IUCN has observer and consultative status at the United Nations, and plays a rolein
the implementation of several international conventions on nature conservation and biodiversity.
o It is funded by governments, bilateral and multilateral agencies, foundations, member organisations and
corporations.

Q: Which among the following is the objectiveof the UNEP's Blue Carbon Initiative?
(a) Development of tidal energy to meetincreasing demand for energy.
(b) Management of coastal ecosystems inorder to ensure that their carbon sequestration are maintained.
(c) Regulate movement of hazardousmaterials through sea routes.
(d) Regulation of activities that contributeto aerosol emissions in the environment.

© APTI PLUS ACADEMY FOR CIVIL SERVICES


Answer: B
Go through the link : http://web.unep.org/

UNEP‟s Blue Carbon Initiative aims to develop a global partnership to advance the sound management of
coastal and marine ecosystems in order to ensure that their carbon sequestration and storage functions are
maintained, and emissions of greenhouse gases are avoided. The initiative aims at achieving the following
overall targets by 2025:
Reversing the current trend of the degradation of marine and coastal habitats and maintaining the
amount of carbon sequestration;
Significantly increasing the area oflue
B Carbon ecosystems under effective ecosystem-
basedmanagement; and
Mobilising US$40 million for protecting and rehabilitating coastal and marine habitats, thus enhancing
their capacity for carbon storage and sequestration.

Q:Which among the following correctlydescribes Earth hour?


(a) It is a movement in which participantsuse public transport only for one hour on a certain day every year .
(b) It is a movement in which the participants switch off the lights for onehour on a certain day every year.
(c) It is a movement in which theparticipating nations prohibit generationof energy by non- renewable
sources forone hour.
(d) It is a movement in which participants clean the streets and roads of the country one a certain day of
every year for one hour.

Answer: B
Study the organization: https://www.worldwildlife.org/

Earth Hour is a worldwide movement for the planet organized by the World Wide Fund for Nature(WWF).
The event is held worldwide annually encouraging individuals, communities, house holds and businesses to
turn off their non-essential lights for one hour, from 8:30 to 9:30 p.m. towards the end of March, as a
symbol for their commitment to the planet.

Q: Consider the following statements with regard to Global Environment Facility(GEF):


1. It was established by World Bank to assist in the protection of the global environment.
2. It serves as a financial mechanism to the United Nations Framework Convention on Climate Change
(UNFCCC).
3. United Nations Environment Program(UNEP) is its implementing Agency.
Which of the statements given above is/are correct?
(a) 1 only
(b) 2 only
(c) 1 and 2 only
(d) 1, 2 and 3

© APTI PLUS ACADEMY FOR CIVIL SERVICES


Answer: D

Study the links: https://www.thegef.org/about-us

All the statements are correct:


The Global Environment Facility is a partnership for international cooperation where 183 countries work
together with international institutions, civil society organizations and the private sector, to address global
environmental issues. The Global Environment Facility was established in October 1991 as a $1billion pilot
program in the World Bank to assist in the protection of the global environment and to promote
environmental sustainable development. The GEF work focuses on seven main areas, including biodiversity,
climate change (mitigation and adaptation), chemicals, international waters, land degradation, sustainable
forest management/REDD+,Ozone layer depletion.
The GEF also serves as financial mechanism for the following four conventions:
Convention on Biological Diversity (CBD)
United Nations Framework Convention on Climate Change (UNFCCC)
Stockholm Convention on Persistent Organic Pollutants (POPs), and
UN Convention to Combat Desertification (UNCCD).

The GEF also provides a separate window for the direct participation of NGOs, local communities, and other
grassroots organizations through the Small Grants Program that is managed by UNDP. World
Bank serves as Trustee and administrator of the GEF Trust Fund. UNEP is an Implementing Agency of the GEF
with the World Bank and the United Nations Development Program (UNDP).

India & GEF:


The Sustainable Land and Ecosystem Management (SLEM) Program: It is a joint initiative of the
Government of India and the Global Environmental Facility (GEF) under the latter's Country partnership
Programme.

Q: Consider the following statements withregard to Intergovernmental Panel onClimate Change (IPCC):
1. It conducts independent research forassessing climate change.
2. It is a joint initiative of World Meteorological Organization (WMO)and United Nations Environment
Programme (UNEP).
3. Its headquarters is located at Geneva.
Which of the statements given above is/arecorrect?
(a) 3 only
(b) 1 and 2 only
(c) 2 and 3 only
(d) 1, 2 and 3

© APTI PLUS ACADEMY FOR CIVIL SERVICES


Answer: C

Study : https://www.ipcc.ch/

The Intergovernmental Panel on Climate Change (IPCC) is the international body for assessing thescience
related to climate change. It was set up in 1988 by the World Meteorological
Organization (WMO) and United Nations Environment Programme (UNEP) to provide policymakerswith
regular assessments of the scientific basis of climate change, its impacts and future risks, and optionsfor
adaptation and mitigation. Thus statement 2 is correct.
The IPCC works by assessing published literature. It does not conduct its own scientific research.
Thusstatement 1 is incorrect.
Its headquarters is located at Geneva, Switzerland. Thus statement 3 is correct.

Q: Which of the following statements is/arecorrect with regard to Globally Important Agricultural Heritage
Systems (GIAHS)?
1. Its aim is to create public awareness and safeguard world agricultural heritage sites.
2. The Food and agricultural organization(FAO) accords the status of GIAHS.
3. Koraput of Odisha is the only recognized GIAHS in India.
Select the correct answer using the codegiven below.
(a) 1 only
(b) 1 and 2 only
(c) 2 and 3 only
(d) 1, 2 and 3

Answer: B
Study : http://www.fao.org/home/en/

GIAHS aims to create public awareness and safeguard world agricultural heritage sites. It was started in
2002 by Food and agricultural organization (FAO) an intergovernmental organization, headquarteredat
Rome which aims at helping world population of food security, hunger elimination and poverty.Hence
statements 1 and 2 are correct.
 Statement 3 is incorrect. Two sites have been recognized as GIAHS in India:
 Koraput of Odisha: This region has rich biodiversity, growing several varieties of paddy, millets,
pulses,oilseeds, vegetables.
 Kashmir Valley, Pampore region: Saffron Heritage Site of Kashmir in India

Q: Consider the following statements with respect to acid rain:


1. Acidified soils tend to have high levels of cadmium, which when absorbed by plants, increases their
toxicity.

© APTI PLUS ACADEMY FOR CIVIL SERVICES


2. Mountain tops are relatively less vulnerable to the effects of acid rain as compared to lowlands.
3. Magnesium, potassium and iron are leached away from soil by acid rains.
Which of the above statement(s) is/are correct?
(a) 3 only
(b) 1 and 3
(c) 1 and 2
(d) 1, 2 and 3

Answer: B
Statement 1 is correct. Acidified soils resulting from acid rain tend to have cadmium, which when absorbed
by plants, increases the toxicity in them. High levels of cadmium in plants are injurious for animals and
human beings.

Statement 2 is incorrect. Acid fog and mist are found on mountain tops, thus increasing the exposure of the
mountainous plants to acidity. Also, the precipitation on mountain tops is more as compared to lowlands, so
they are more exposed to acidic preparation.

Statement 3 is correct. Nutrients like calcium, potassium, magnesium and iron, which are essential for plant
growth, are leached away from the soil by acid rains, thus making it nutrient deficient.

Q: Which of the following are the effects of increase in UV radiation due to the depletion of the Ozone
layer?
1. Temperature changes and rainfall failure
2. Genetic disorders
3. Reduction in chlorophyll content of plants
4. Decrease in the productivity of continental shelves
Select the correct answer using the code given below.
(a) 1 and 2 only
(b) 2 and 3 only
(c) 1, 3 and 4 only
(d) 1, 2, 3 and 4

Answer: D

Depletion of the Ozone layer will lead to an increase in the Ultraviolet radiations received by the Earth.
This is likely to cause both direct and indirect harmful effects.

Direct effects: The temperature in the stratosphere increases due to absorption of the UV-rays by the Ozone
layer. Depletion of the layer will cause temperature changes and rainfall failures on the earth; increased
harmful radiations will lead to reduction in the immunity and increased susceptibility to diseases in plants

© APTI PLUS ACADEMY FOR CIVIL SERVICES


and animals, increased incidence of cataracts and cancers, genetic disorders which will ultimately affect the
heredity.
Indirect effects: Reduction in growth of plants, reduction in chlorophyll content, increase in mutations due
to exposure to UV-radiation, the UV-rays have the potential to disturb the ecological balance in marine
ecosystems. Young cells and larvae, green algae, fish and other animals in the continental shelves get
affected due to increased exposure to UV radiation.
All the statements are correct.

Q: Which of the following gases is/are associated with acid rain?


1. Sulphur Dioxide
2. Nitrogen Oxides
3. Methane
4. Ozone
Select the correct answer using the code given below.
(a) 1 and 2 only
(b) 1, 2 and 3 only
(c) 2, 3 and 4 only
(d) 1 and 4 only

Answer: A
Acid rain results when sulfur dioxide (SO2) and nitrogen oxides (NOX) are emitted into the atmosphere and
transported by wind and air currents. The SO2 and NOX react with water, oxygen and other chemicals to
form sulfuric and nitric acids. These then mix with water and other materials before falling to the ground.
While a small portion of the SO2 and NOX that cause acid rain is from natural sources such as volcanoes,
most of it comes from the burning of fossil fuels.

Q: With reference to acidification of ocean, consider the following statements:


1. It occurs due to the high absorption of nitrogenous based acidic compounds.
2. Productivity of corals will increase because of the increase in nitrogenous nutrients.
3. Introduction of sea grasses can reduce the impact of acidification.
Which of the statements given above is/are correct?
(a) 1 and 2 only
(b) 3 only
(c) 2 and 3 only
(d) 1, 2 and 3

Answer: B

© APTI PLUS ACADEMY FOR CIVIL SERVICES


o When carbon dioxide dissolves in this ocean, carbonic acid is formed. This leads to higher acidity, mainly
near the surface, which has been proven to inhibit shell growth in marine animals and is suspected as a
cause of reproductive disorders in some fish.
o As human activities pump ever-greater quantities of carbon dioxide into the atmosphere, our ocean
waters absorb that gas, which results in ocean acidification.
o Shell-forming animals including corals, oysters, shrimp, lobster, many planktonic organisms, and even
some fish species could be gravely affected. This is because numerous marine organisms such as corals,
mollusks, crustaceans and seaurchins rely on carbonate ions to form their calcareous shells or skeletons in a
process known as calcification.
o Ocean acidification is expected to impact ocean species to varying degrees. Photosynthetic algae and
seagrasses may benefit from higher CO2 conditions in the ocean, as they require CO2 to live just like plants
on land.
o Solution to reduce acidification: Seagrass, an organism with amazing properties: this plant absorbs carbon
dioxide, uses it to produce more seagrass and changes the very chemistry of the water around it, reducing
the impact of ocean acidification.

Q: Which of the following are the possible effects of global warming?


1. The rate of decomposition of organic matter will increase
2. Sowing period will decrease
3. Faster deterioration of soil quality
Select the correct answer using the codes given below.
(a) 3 only
(b) 1 and 2 only
(c) 2 and 3 only
(d) 1, 2 and 3

Answer: D
Global warming and the resulting change in climatic conditions have the potential to affect the agro
ecosystems in the following areas:
o Increase in temperature will increase evaporation.
o Cultivable areas will be enlarged.
o Soil water will become insufficient because of excessive evaporation and transpiration.
o Water and soil temperatures will rise.
o Sowing period will shorten.
o The incidence of insect pests, diseases and weeds will become higher. The activities of microorganisms
will increase. The decomposition of organic matter and fertilizers will be promoted.
o The deterioration of soil quality will be quickened.

Q: Which of the following activities increases the concentration of Carbon Dioxide in the atmosphere?

© APTI PLUS ACADEMY FOR CIVIL SERVICES


1. Decomposition of limestone during the manufacture of cement
2. Photosynthesis by plants
3. Volcanic eruptions
4. Deforestation
Select the correct answer using the code given below.
(a) 1, 3 and 4 only
(b) 2 and 3 only
(c) 1, 2 and 4 only
(d) 1 and 4 only

A
Statement 1 is correct. Decomposition of limestone during the manufacture of cement releases CO2 in the
atmosphere.
Statement 2 is incorrect. Photosynthesis by plants releases oxygen to the atmosphere and absorbs CO2.
Statement 3 is correct. Volcanic eruption releases CO2 to the atmosphere.
Statement 4 is correct. Deforestation increased the concentration of CO2 in the atmosphere.

Q: Concentration of which of the following gases in atmosphere result in greenhouse gas effect?
1. Stratospheric Ozone
2. Water vapour
3. Hydro-Flouro Carbon
4. Methane
Select the correct answer using the code given below.
(a) 1, 2 and 4 only
(b) 2, 3 and 4 only
(c) 1, 2, 3 and 4
(d) 1 and 3 only

Answer:B
While Stratospheric Ozone is not a Green house gas, Tropospheric Ozone is a Green house gas.
Water vapour traps radiation coming from land and thus increases temperature in the atmosphere
therefore it is a GHG.
Hydro-flouro Carbon and Methane are GHGs.

© APTI PLUS ACADEMY FOR CIVIL SERVICES


Q: Consider the following pairs:
Notified Area: Declared under the act
1. Marine Protection Area: Biological Diversity Act 2002
2. Coastal Regulation Zone: Environment (Protection) Act, 1986
3. Biodiversity Heritage Site: Wild Life (Protection) Act, 1972
Which of the pairs given above is/are correctly matched?
(a) 2 only
(b) 2 and 3 only
(c) 1 and 3 only
(d) 1, 2 and 3

Answer: A
Marine protected area (MPA) is notified under Wildlife Protection act while Coastal Regulation Zone is
notified under Environmental Protection act. Biodiversity Heritage Site is notified under Biodiversity
Act.

Q: As per the Wildlife Protection Act 1972, the Chief wildlife warden may permit hunting of wildlife animals
under which of the following conditions?
1. A wild animal has become dangerous to human life.
2. A wild animal has become diseased beyond recovery
3. Hunting of snakes for collecting venom to prepare life saving drugs.
Select the correct answer using the code given below.
(a) 1 and 3 only
(b) 1 only
(c) 2 and 3 only
(d) 1, 2 and 3

Answer: D
All the given acts are permissible under various provisions of the acts.
According to Section 11 of Wildlife Protection Act, 1972: the Chief Wildlife Warden may permit hunting
o if he is satisfied that any wild animal specified in Sch. 1 has become dangerous to human life or is so
disabled or diseased as to be beyond recovery, by order in writing and stating the reasons therefore, permit
any person to hunt such animal or cause animal to be hunted.
o The killing or wounding in good faith of any wild animal in defence of oneself or of any other person shall
not be an offense. Similarly Section 12 allows hunting snakes for collection of venoms for medical purposes.

Q: Consider the following statements:


1. While a central legislation is needed for altering the boundaries of a National Park, a resolution is passed
by state legislature for altering boundaries of a Sanctuary.
2. While no human activity is allowed inside a national park, limited activities are permitted within the
sanctuary.

© APTI PLUS ACADEMY FOR CIVIL SERVICES


Which of the statements given above is/are correct?
(a) 1 only
(b) 2 only
(c) Both 1 and 2
(d) Neither 1 nor 2

Answer: B
o No alteration of the boundaries of a National Park shall be made except on the resolution passed by the
legislature of the State. The same is true for a Wildlife Sanctuary. Therefore, statement 1 is not correct.
o While no human activity is allowed inside a national park, limited activities are permitted within the
sanctuary. Therefore, statement 2 is correct.

Q: Which of the following provisions is/are part of e-Waste Management Rules, 2016?
1. Inclusion of CFL under the purview of e-Waste Management Rules.
2. 'Deposit Refund Scheme' for sale of electrical and electronic equipment.
Select the correct answer using the code given below:
(a) 1 only
(b) 2 only
(c) Both 1 and 2
(d) None of the above

Answer: c
Statement 1 is correct. Compact Fluorescent Lamp (CFL) and other mercury containing lamp brought under
the purview of rules.
Statement 2 is correct. Deposit Refund Scheme has been introduced as an additional economic instrument
wherein the producer charges an additional amount as a deposit at the time of sale of the electrical and
electronic equipment and returns it to the consumer along with interest when the end-of-life electrical and
electronic equipment is returned.

Q: Consider the following statements regarding Solid Waste Management Rules, 2016:
1. The rules have been made applicable beyond municipal areas to include urban agglomerations and census
towns.
2. Responsibility of waste segregation has been placed on waste generators.
3. Waste generators will have to pay 'User Fee' to waste collector.
Which of the statements given above is/are correct?
(a) 1 and 2 only
(b) 1 and 3 only
(c) 2 only

© APTI PLUS ACADEMY FOR CIVIL SERVICES


(d) 1, 2 and 3

Answer: D
The Environment Ministry has revised Solid Waste Management Rules after 16 years. Following are it's
salient features:
Rules are now applicable beyond municipal areas and will extend to urban agglomerations, census towns,
notified industrial townships, areas under the control of Indian Railways, airports, etc. Hence, statement 1 is
correct.
The responsibility of genera tors has been introduced to segregate waste into three categories – Wet
(Biodegradable), Dry (Plastic, Paper, metal, wood, etc.) and domestic hazardous wastes (diapers, napkins,
empty containers of cleaning agents, mosquito repellents, etc.). Hence, statement 2 is correct.
The generator will have to pay „User Fee‟ to the waste collector and a „Spot Fine‟ for littering and
nonsegregation. Hence, statement 3 is correct.
Waste processing facilities will have to be set up by all local bodies having 1 millio
n or more population
within two years.
Bulk and institutional generators, market associations, event organizers and hotels and restaurants have
been made directly responsible for segregation and sorting the waste and manage in partnership with local
bodies.
Food waste from hotels and restaurants is utilized for composting /biomethanation.

Q.6: With reference to Clean Development Mechanism (CDM) of the Kyoto Protocol, consider the following
statements:
1. The CDM allows emission-reduction projects in developing countries to earn certified emission reduction
(CER) credits.
2. Such CERs cannot be traded and sold, and used by industrialized countries to meet a part of their emission
reduction targets.
3. It is the main source of income for the UNFCCC Adaptation Fund.
Which of the statement(s) given above is/are incorrect?
a) 2 only
b) 1 and 3 only
c) 2 and 3 only
d) None of the above

Answer: a
Explanation: Statement 2 is incorrect. Certified Emission Reduction (CER) can be held by governmental and
private entities on electronic accounts with the UN.
CERs can be purchased from the primary market (purchased from an original party that makes the
reduction) or secondary market (resold from a marketplace).
So, such CERs can be traded and sold, and used by industrialized countries to meet a part of their emission
reduction targets.

Q: Consider the following statements regarding Basel Convention:


1. It aims to regulate the disposal and transboundary movement of hazardous waste.
2. India is a signatory to the convention.

© APTI PLUS ACADEMY FOR CIVIL SERVICES


Which of the statements given above is/are correct?
(a) 1 only
(b) 2 only
(c) Both 1 and 2
(d) Neither 1 nor 2

Answer: C
Statement 1 is Correct. The Basel Convention on the control of Transboundary movements of hazardous
wastes and their disposal first came into force in 1992. The Convention puts an onus on exporting countries
to ensure that hazardous wastes are managed in an environmentally sound manner in the country of
import.
Statement 2 is correct. India is a party and a signatory to the convention.

Q: Which of the following is/are mandated by the Convention on Biological Diversity (CBD)?
1. Involvement of locals with traditional knowledge in conservation efforts.
2. Equitable sharing of benefits arising from the utilisation of traditional knowledge with the locals.
Which of the statements given above is/are correct?
(a) 1 only
(b) 2 only
(c) Both 1 and 2
(d) Neither 1 nor 2

Answer: C
Statement 1 is correct. In the context of CBD, "Traditional knowledge" is employed to mean knowledge,
innovations and practices of indigenous and local communities embodying traditional lifestyles relevant for
the conservation and sustainable use of biological diversity.
Each state must respect, preserve and maintain knowledge, innovations and practices of indigenous and
local communities embodying traditional lifestyles relevant for the conservation and sustainable use of
biological diversity and promote their wider application with the approval and involvement of the holders of
such knowledge, innovations and practices and encourage the equitable sharing of the benefits arising from
the utilization of such knowledge innovations and practices. Therefore statement 2 is also correct.

Q: Consider the following statements regarding United Nations Framework Convention on Climate Change
(UNFCCC):

© APTI PLUS ACADEMY FOR CIVIL SERVICES


1. It was adopted at the "Rio Earth Summit" in 1992.
2. It is a legally binding as it sets mandatory limits on greenhouse gas emissions.
3. Its ultimate objective is to eliminate greenhouse gas in the atmosphere.
Which of the statements given above is/are correct?
(a) 1 only
(b) 1 and 3 only
(c) 2 and 3 only
(d) 1, 2 and 3

Answer: A
o The UNFCCC is a "Rio Convention", one of three adopted at the "Rio Earth Summit" in 1992. Its sister Rio
Conventions are the UN Convention on Biological Diversity and the Convention to Combat Desertification.
o The treaty itself is not legally binding as it does not set mandatory limits on greenhouse gas emissions for
individual countries and doesn't contain any enforcement mechanisms.
o The ultimate objective of the Convention is to "stabilise (and not eliminate) greenhouse gas
concentrations in the atmosphere at a level that will prevent dangerous human interference with the
climate system." It states that "such a level should be achieved within a time-frame sufficient to allow
ecosystems to adapt naturally to climate change, to ensure that food production is not threatened, and to
enable economic development to proceed in a sustainable manner."

Q: Consider the following pairs:


Conference Objective
1. Saving the Ozone Layer conference: First global conference related to the Ozone layer
2. Helsinki conference: Revision of Montreal protocol
3. Vienna conference: Total ban on Ozone depleting CFCs
Which of the pairs given above is/are correctly matched?
(a) 1 and 2 only
(b) 2 only
(c) 2 and 3 only
(d) 1 and 3 only

Answer: B
The Vienna conference was the first global conference on the depletion of the ozone layer, held in Vienna,
Austria in 1985. It was held after a hole as large as the size of the US was discovered in the ozone layer.At
this conference, Vienna convention for the protection of the Ozone layer was agreed.

Montreal protocol was a landmark convention where major steps were charted-out to be taken, to reduce
CFC production. While the developed countries decided to reduce CFC production step by step, some
relaxations were considered for developing countries like India and China. It was agreed that the extra costs
for developing alternatives for CFCs would be undertaken by the developed countries.

The Saving the Ozone Layer Conference was an international meet, which was organised jointly by
UNEP and the British Government in 1989, to draw attention to the global problem created by the
developed world, which was trying to dictate its terms to the developing countries for CFCs pollution. It
stressed upon the final withdrawal of all Ozone depleting CFCs and other chemicals.

© APTI PLUS ACADEMY FOR CIVIL SERVICES


The Helsinki conference was held in 1989, to revise the Montreal Protocol. As many as 80 countries agreed
to have a total ban by 2000 on chemicals that cause ozone depletion. The agreement for CFC elimination is a
major step towards environmental protection.

Q: In the context of Second commitment period of the Kyoto Protocol, which of the following statements
is/are correct?
1. Amendments to Kyoto Protocol for the second commitment 2013- 2020 period were successfully adopted
at Doha.
2. It deals with containing the emission of Green House Gases (GHGs).
3. India is yet to ratify the Second Commitment Period of the Kyoto Protocol.
Select the correct answer using the code given below.
(a) 1 only
(b) 2 only
(c) 1 and 2 only
(d) 1, 2 and 3

Answer: C
The Kyoto Protocol was adopted in 1997 and the 1st commitment period was from 2008 -
2012. At Doha in 2012, the amendments to Kyoto Protocol for the 2nd commitment period (the Doha
Amendment) were successfully adopted for the period 2013- 2020. Hence statement 1 is correct.
Developed countries have already started implementing their commitments under the 'opt-in' provisions of
the Doha Amendment.
Second commitment period of the Kyoto Protocol on containing the emission of Green House Gases
(GHGs) (statement 2 is correct) was ratified by India recently on 24 January 2017. Hence statement 3 is
incorrect.

Q: According to a recent report by TRAFFIC, India accounts for 29% of black spotted turtles seized from
countries in South Asia. With reference to this consider the following statements.
1. Black Spotted Turtles are identified as Endangered by IUCN.
2. TRAFFIC is an international network which monitors trade in wildlife.
3. TRAFFIC is a strategic alliance of the World Wide Fund for Nature and IUCN.
Which of the above statement(s) is/are correct?
(a) 2 only
(b) 1 and 3 only
(c) 2 and 3 only
(d)All of the Above

Answer- c
Statement 1 is incorrect: They are classified as vulnerable by the IUCN.

© APTI PLUS ACADEMY FOR CIVIL SERVICES


Statement 2 is correct: TRAFFIC, the Wildlife Trade Monitoring Network, is the leading non-governmental
organization working globally on the trade of wild animals and plants in the context of both biodiversity and
sustainable development.

Statement 3 is correct: It was founded in 1976 as a strategic alliance of the World Wide Fund for Nature
(WWF) and the International Union for the Conservation of Nature (IUCN).

Q: Consider the following statements regarding Stockholm convention:


1. It regulates the trans-boundary movement of all hazardous waste.
2. India is yet to ratify the convention.
3. Dichloro diphenyl trichloro ethane (DDT), a widely used pesticide in India, has been listed under the
convention.
Which of the statements given above is/are correct?
(a) 1 and 3 only
(b) 1 and 2 only
(c) 3 only
(d) 1, 2 and 3

Answer: C
The Stockholm Convention on Persistent Organic Pollutants (POPs) is a global treaty to protect human
health and the environment from chemicals that remain intact in the environment for long periods, become
widely distributed geographically, accumulate in the fatty tissue of humans and wildlife, and have harmful
impacts on human health or on the environment. Thus statement 1 is incorrect as it covers POPs only.
Statement 2 is incorrect. India has ratified the convention in 2006.
Statement 3 is correct. DDT is now classified as a persistent organic pollutant (POP) under the stockholm
convention as it accumulates in fatty tissues, and can travel long distances in the upper atmosphere.

Q: Consider the following statements with regard to Bonn convention:


1. It is an environmental treaty under UNEP.
2. Its objective is to promote conservation of migratory species.
3. It covers only terrestrial and avian species, not marine species.
Which of the statements given above is/are correct?
(a) 1 and 3 only
(b) 1 and 2 only
(c) 2 only
(d) 1, 2 and 3

Answer: B

© APTI PLUS ACADEMY FOR CIVIL SERVICES


Bonn convention (Convention on the Conservation of Migratory Species of Wild Animals (CMS)) is an
environmental treaty under the aegis of the United Nations Environment Programme.
The Bonn Conventionims a to conserve terrestrial, marine and avian migratory species throughout their
range. Hence Statement 1 and 2 are correct and statement 3 is incorrect.
CMS acts as a framework Convention. The agreements may range from legally binding treaties (called
Agreements) to less formal instruments, such as Memoranda of Understanding, and can be adapted to the
requirements of particular regions. Thus the convention is not itself legally binding.
Migratory species threatened with extinction are listed on Appendi x I of the Convention. CMS Parties
strive towards strictly protecting these animals, conserving or restoring the places where they live,
mitigating obstacles to migration and controlling other factors that might endanger them.
Migratory species that need or would significantly benefit from international co-operation are listed in
Appendix II of the Convention.

Q: Consider the following statements regarding CITES:


1. It is legally binding on member countries.
2. India is a member country to the convention.
3. It prohibits trade in all the species which are classified as Threatened by IUCN Red list only.
Which of the statements given above are correct?
(a) 1 and 3 only
(b) 1 and 2 only
(c) 2 and 3 only
(d) 1, 2 and 3

Answer: B
CITES (the Convention on International Trade in Endangered Species of Wild Fauna and Flora) is an
international agreement between governments. Its aim is to ensure that international trade in specimens of
wild animals and plants does not threaten their survival.
CITES is legall
y binding on the Parties – in other words they have to implement the Convention – it does
not take the place of national laws. Rather it provides a framework to be respected by each Party, which has
to adopt its own domestic legislation to ensure that CITES is implemented at the national level. India is a
member country to the convention. Hence Statement 1 and 2 are correct.
 Statement 3 is incorrect. CITES maintain their own separate list. The species covered may or may not not
be in the IUCN red list.
The species covered by CITES are listed in three Appendices, according to the degree of protection they
need.
Appendix I includes species threatened with extinction. Trade in specimens of these species is permitted
only in exceptional circumstances. Thus it does not prohibit the trade completely. The statement 3 is
incorrect.
Appendix II includes species notecessarily
n threatened with extinction, but in which trade must be
controlled in order to avoid utilization incompatible with their survival.
Appendix III: This Appendix contains species that are protected in at least one country, which has asked
other CITES Parties for assistance in controlling the trade.

© APTI PLUS ACADEMY FOR CIVIL SERVICES


Q: Which of the following conventions deal with the handling and movement of hazardous waste material?
1. Basel Convention
2. Agenda 21
3. Rotterdam Convention
Select the correct answer using the code given below.
(a) 1 and 2 only
(b) 3 only
(c) 2 and 3 only
(d) 1 and 3 only

Answer: D
Both Basel Convention and Rotterdam Convention are related to handling and movement of hazardous
waste material. Hence answer is (d).
Basel Convention: is an international treaty that was designed to reduce the movements of hazardous
waste between nations, and specifically to prevent transfer of hazardous waste from developed to less
developed countries (LDCs). It does not, however, address the movement of radioactive waste.
Rotterdam Convention is a multilateral treaty to promote shared responsibilities in relation to
importation of hazardous chemicals. promotes open exchange of information and calls on exporters of
hazardous chemicals to use proper labeling, include directions on safe handling, and inform purchasers of
any known restrictions or bans.
Agenda 21, an outcome of Earth summit 1992, deals with sustainable development.

Q: Consider the following statements regarding 'Montreux Record':


1. The wetlands included in the Record are under ecological threat due to human interference.
2. It is maintained as part of the Ramsar List.
3. No wetlands of India are in the 'Montreux Record'.
Which of the statements given above is/are correct?
(a) 1 and 2 only
(b) 2 only
(c) 1 and 3 only
(d) 1, 2 and 3

Answer: A
The Convention on Wetlands, signed in Ramsar, Iran, in 1971, is an intergovernmental treaty which
provides the framework for national action and international cooperation for the conservation and wise use
of wetlands and their resources.
Montreux Record under the Convention is a register of wetland sites on the List of Wetlands of
International Importance where changes in ecological character have occurred, are occurring, or are likely to
occur as a result of technological developments, pollution or other human interference. It is maintained as
part of the Ramsar List. Hence statement 1 and 2 are correct.
Currently, two wetlands of India are in Montreux record viz. Keoladeo National Park, Rajasthan and

© APTI PLUS ACADEMY FOR CIVIL SERVICES


Loktak Lake, Manipur. Further, Chilka lake was placed in the record but was later removed from it. Hence
Statement 3 is incorrect.

Q: Consider the following statements with respect to Cartagena Protocol:


1. It establishes procedures for regulating the import and export of Living Modified Organisms.
2. It regulates upon access and benefit sharing of genetic resources.
3. India is a party to the convention.
Which of the statements given above is/are correct?
(a) 1 only
(b) 2 and 3 only
(c) 1 and 3 only
(d) 1, 2 and 3

Answer: C
The Cartagena Protocol on Bio safety to the Convention on Biological Diversity is an international
agreement which aims to ensure the safe handling, transport and use of living modified organisms
(LMOs) resulting from modern biotechnology that may have adverse effects on biological diversity, taking
also into account risks to human health. Hence statement 1 is correct. It was adopted on 29 January 2000
and entered into force on 11 September 2003.
It was negotiated under the aeg is of the Convention on Biological Diversity (CBD). The protocol was
adopted on 29th January 2000. With Palau acceding to the Protocol, the required number of 50 instruments
of ratification /accession /approval /acceptance by countries was reached in May 2003. The
Protocol entered into force on 11 September 2003. Currently 169 countries are Parties to the Protocol.
India is a party to the United Nations Convention on Biological Diversity signed at Rio de Janeiro on the
5th day of June, 1992 which came into force on the 29th December, 1993. Hence statement 3 is correct.
The Nagoya Protocol, a supplementary agreement to the Convention on Biological Diversity, provides a
legal framework for implementing that objective of access and benefit sharing of genetic resources. Hence
statement 2 is incorrect.

© APTI PLUS ACADEMY FOR CIVIL SERVICES

Potrebbero piacerti anche